Как найти полное комплексное сопротивление

ads

Одним из способов расчета цепей переменного тока является комплексный, или еще как говорят, символический метод расчета. Этот метод применяется при анализе схем с гармоническими ЭДС, напряжениями и токами. В результате решения получают комплексное значение токов и напряжений, используя для решения любые методы (эквивалентных преобразований, контурных токов, узловых потенциалов и т.п.). Но для начала необходимо иметь понятие, в каких именно формах может представляться синусоидальная величина. 1. Одна из форм представления – это вращающийся вектор (см. рис.1):

Вращающийся вектор

Рис.1. Вращающийся вектор

С помощью рисунка ясно видно, как с течением времени меняется значение синусоидальной величины. В нашем случае – это величина а на графике, которая может быть, например, входным напряжением. Величина имеет некоторое начальное значение при t = 0 при начальной фазе φ

имеет положительное максимальное значение при угле ωt3, когда при времени t3 сумма ωt3φ = 90° и соответственно,

имеет отрицательное максимальное значение при угле ωt7, когда при времени t7 сумма углов ωt7φ = 270° и, соответственно,

и имеет два нулевых значения при ωtn + φ = 0, когда ωtn = —φ (на рис.1 эта область не показана и находится слева от начала координат)

и тогда

и имеет нулевое значение при угле ωt11, когда при времени t11 сумма ωt11φ = 360° и соответственно,

Именно по такому закону и меняется привычное нам переменное напряжение 220 В, изменяясь по синусоидальному закону от  значения  0 В до максимальных 311 В и обратно.

2. Другая форма представления – это комплексное число. Чтобы представить ранее рассмотренную форму представления синусоидальной величины, которая имеет некоторую начальную фазу φ, создают комплексную плоскость в виде графика зависимости двух величин (рис.2)

Комплексное число на комплексной плоскости

Рис.2. Комплексное число на комплексной плоскости

Длина вектора Am на такой комплексной плоскости равна амплитуде (максимальному значению) рассматриваемой величины. С учетом начальной фазы φ такое число записывают как .

На практике при использовании для расчетов символического (комплексного) метода расчета используют для некоторых удобств не амплитудное значение величины, а так называемое действующее значение. Его величина в корень из двух раз меньше амплитудного и обозначается без индекса m, т.е. равна

действующее значение

На рисунке выше этот вектор также показан.
Например, при том же нашем напряжении в сети, максимальное значение синусоидально изменяющегося напряжения равно 311 В, а действующее значение, к значению которого мы привыкли

Действующее значение напряжения

При работе с комплексными числами и расчетов применяют различные формы записи комплексного числа. Например, при сложении комплексных чисел удобнее использовать алгебраическую форму записи таких чисел, а при умножении или делении – показательную форму записи. В некоторых случаях пишут тригонометрическую форму.
Итак, три формы записи комплексного числа:

1) показательная форма в виде

Показательная форма комплексного числа

2) тригонометрическая форма в виде

Тригонометрическая форма комплексного числа

3) алгебраическая форма

Алгебраическая форма комплексного числа

где ReA — это действительная составляющая комплексного числа, ImA — мнимая составляющая.

Например, имеем комплексное число в показательной форме вида

в тригонометрической форме записи это запишется как

при подсчете получим число, плавно переходящее в алгебраическую форму с учетом того, что

В итоге получим

где

При переходе от алгебраической формы к показательной комплексное число вида

переходит к показательному виду  по следующим преобразованиям

а угол

Таким образом, и получим

Перейдем к рассмотрению несложных примеров использования  символического, или по-другому, комплексного метода расчета электрических цепей. Составим небольшой алгоритм комплексного метода:

      • Составить комплексную схему, заменяя мгновенные значения ЭДС, напряжений и токов их комплексным видом
      • В полученной схеме произвольно выбирают направления токов в ветвях и обозначают их на схеме.
      • При необходимости составляют комплексные уравнения по выбранному методу решения.
      • Решают уравнения относительно комплексного значения искомой величины.
      • Если требуется, записывают мгновенные значения найденных комплексных величин.

Пример 1. В схеме рис.3 закон изменения ЭДС e = 141sin*ωt. Сопротивления R1 = 3 Ом, R2 = 2 Ом, L = 38,22 мГн, С = 1061,6 мкФ. Частота f = 50 Гц. Решить символическим методом. Найти ток и напряжения на элементах. Проверить 2-ой закон Кирхгофа для цепи.

Схема с последовательным соединением элементов

Рис.3. Схема с последовательным соединением элементов

Составляем комплексную схему, обозначив комплексные токи и напряжения (рис.4):

Схема с комплексными обозначениями

Рис.4. Схема с комплексными обозначениями

По закону Ома ток в цепи равен

Закон ома в комплексной форме

где U — комплексное входное напряжение, Z — полное сопротивление всей цепи. Комплекс входного напряжения находим как

Пояснение: здесь начальная фаза  φ = 0°, так как  общее выражение для мгновенного значения напряжение вида при  φ = 0° равно

Соответственно, комплекс входного напряжения в показательной форме запишется как

Полное комплексное сопротивление цепи в общем виде

Находим комплексное сопротивление индуктивности

Находим комплексное сопротивление емкости

Соответственно, общее комплексное сопротивление цепи

Ток в цепи

Комплексные напряжения на элементах

Проверяем второй закон Кирхгофа для замкнутого контура, т.е. должно выполняться равенство

Проверяем

С небольшим расхождением из-за округлений промежуточных вычислений всё верно.

Пример 2. В электрической цепи (рис.5) однофазного синусоидального тока, схема и параметры элементов которой заданы для каждого варианта в таблице, определить:
1)  полное сопротивление электрической цепи и его характер;
2)  действующие значения токов в ветвях;
3) показания вольтметра и ваттметра;

    1. Исходные данные: Е = 220 В, f = 50 Гц, L1 = 38,2 мГн, R2 = 6 Ом, С2 = 318 мкФ, L2 = 47,7 мГн, R3 = 10 Ом, С3 = 300 мкФ.

Рис.5.Цепь однофвзного синусоидального тока

Решение:
1.  Находим комплексные сопротивления ветвей и всей цепи:
Учитываем, что

Комплексное сопротивление первой ветви:

Комплексное сопротивление второй ветви:

Комплексное сопротивление третьей ветви:

Общее сопротивление цепи

Откуда

— нагрузка носит активно-индуктивный характер

2. Находим действующие значения токов в ветвях:

Рис.6. Схема с обозначенными комплексными токами

Действующие значения, соответственно,

3. Определим показания приборов:
Вольтметр подключен по схеме параллельно источнику питания. Соответственно его показание равно:
U=220 В
Ваттметр включен токовой обмоткой в разрыв третьей ветви, а обмоткой напряжения также к выводам третьей ветви, измеряя, таким образом,  активную мощность третьей ветви. Эта мощность равна мощности на сопротивлении R3. Его показания:

Комплексное сопротивление элемента (участка цепи)

Под комплексным сопротивлением понимают
отношения комплексной амплитуды входного
напряжения к комплексной амплитуде
входного тока:

.
(1.6)

где Z
–модуль
комплексного сопротивления, φ=ψu
ψi
– начальная фаза или аргумент комплексного
сопротивления; R
активного
сопротивления, X–
реактивному сопротивлению, причем
Z=(R2+X2)1/2,
а φz(ω)=ψuψi
=arctg(X/R).

По виду записи
комплексного сопротивления можно судить
о характере участка цепи: Z=R
– активное (резистивное) сопротивление;
Z=R+jX —
активно-индуктивное сопротивление; Z=R
– j X
 —
активно-емкостное


комплексная проводимость, величина,
обратная комплексному сопротивлению:

Метод комплексных амплитуд состоит
в следующем:

1) исходная
схема электрической цепи заменяется
комплексной схемой замещения, в которой:

а) все пассивные элементы заменяются
их комплексными сопротивлениями, как
показано на рис. 4.27.

б)
все токи и напряжения в схеме заменяются
их комплексными амплитудами, т.е.х(t) =Xm
cos(0t
x) Xm
=Xm
ejx.


R

ZR=R

C

ZC=1/(jC)

L

ZL=jL


Рис. 4.27

2)
Расчет электрической цепи сводится к
составлению уравнений состояния цепи
на основе законов Ома и Кирхгофа в
комплексной форме и нахождениюкомплексных
амплитуд токов или напряжений на
интересующих нас участках цепи, т.е.Ym
Ym
ejy.

3) Запись окончательного решения состоит
в замене рассчитанных комплексных
амплитуд на гармонические функции
времени, т.е.

Ym=Ym
jy

y(t) =Ym
cos(0t – y).

Пример 5. Алгоритм
метода рассмотрим на примере анализа
цепи, структура которой приведена на
рис. 4.29.

Рис. 4.29. RLC-цепь
второго порядка

На вход цепи
подается синусоидальное воздействие
.
Параметры воздействия и элементов цепи
известны:Um=1
В, ω =1 с-1
, φ u=900
, R=1
Ом, L=1
Гн, C=1
Ф. Требуется определить токи и напряжения
ветвей, построить векторную диаграмму.

Решение.

  1. Представим
    воздействие в комплексной форме:

.

  1. Построим схему
    замещения цепи в частотной области,
    заменив элементы цепи комплексными
    двухполюсниками, как это показано на
    рис. 4.30.

Рис. 4.30. Схема
замещения цепи в частотной области

3. Произведем расчет
реакций (токов и напряжений) в комплексной
области. При этом можно воспользоваться
законами Кирхгофа и Ома в комплексной
форме, а также известными методами
расчета резистивных цепей:

,
,,

,

,
,

,

,
.

  1. Построим векторную
    диаграмму для токов и напряжений в
    цепи. Для этого на комплексной плоскости
    откладываются в соответствующем
    масштабе найденные токи и напряжения,
    как показано на рис. 4.31.

Рис. 4.31. Векторная
диаграмма

 Построение
векторной диаграммы, как правило,
является конечным результатом решения
подобных задач. Векторная диаграмма
показывает амплитуду и начальную фазу
любого тока или напряжения. При
необходимости записать временную
функцию тока или напряжения, это всегда
можно сделать, имея векторную диаграмму.
Например, напряжение на L-элементе
имеет амплитуду
,
а начальную фазу 1350,
значит, во временной области это
напряжение можно записать так:

.

Пример 2.

Активное сопротивление
катушки Rк=6
Ом, индуктивное Xl=10
Ом. Последовательно с катушкой включено
ативное сопротивление R=2Ом
и конденсатор сопротивлением хс=4
Ом (рис.2,а). К цепи приложено напряжение
U=50В
( действующее значение). Определить :1)
полное сопротивление цепи;2)ток;3)коэффициент
мощности;4)активную, реактивную и полную
мощности;5) напряжения на каждом
сопротивлении. Начертите в масштабе
векторную диаграмму цепи.

Решение:

1.Определяем полное
сопротивление цепи

2.Определяем ток

3.Определяем
коэффициент мощности цепи

по таблицам Брадиса
находим =36050’
. Угол сдвига фаз 
находим по синусу во избежание потери
знака угла ( косинус является четной
функцией)

4.Определяем
активную мощность цепи

или

Здесь

5.Определяем
реактивную мощность цепи

6.Определяем
активную мощность цепи

или

7.Определяем падение
напряжения на сопротивлениях цепи

;
;;

Построение векторной
диаграммы начинаем с выбора масштаба
для тока и напряжения. Задаемся масштабом
по току : в 1см – 1,0А и масштабом по
напряжению : 1см- 10В. Построение векторной
диаграмм ( рис.2,.б) начинаем с вектора
тока, который откладываем по горизонтали
в масштабе

Вдоль вектора тока
откладываем векторы падения напряжения
на активных сопротивления URк
и UR:

Из конца вектора
UR
откладываем
в сторону опережения вектора тока на
900
вектор падения напряжения UL
на индуктивном сопротивлении длиной
.Из
конца вектораUI
откладываем в сторону отставания от
вектора тока на 900
вектор падения напряжения на конденсаторе
UC
длиной
.
Геометрическая сумма векторовURк,
UR,
UL
и UC
равна полному напряжению U,
приложенному к цепи .

Пример 3.

На рис. 3,а задана
векторная диаграмма для неразветвленной
цепи, ток I
и падения напряжений на каждом
сопротивлении ( U1,
U2
и т.д.) Определить характер и величину
каждого сопротивления, начертить
эквивалентную схему цепи, вычислить
приложенное напряжение и угол сдвига
фаз .

Решение:

1.Из векторной
диаграммы следует, что напряжение U1
отстает от тока на угол 900.
Следовательно, на первом участке
включен конденсатор, сопротивление
которого

Вектор напряжение
на втором участке U2
направлен параллельно вектору тока,
т.е. совпадает с ним по фазе. Значит, на
втором участке включено активное
сопротивление

Вектор напряжения
на третьем участке U3
опережает вектор тока на угол 900,
что характерно для индуктивности,
сопротивление которой

На четвертом
участке включено активное сопротивление

Эквивалентная
схема цепи приведена на рис. 3, б.

2.Из векторной
диаграммы определяем значение приложенного
напряжения и угол сдвига фаз:

.

Пример:

К электрической
цепи, рис. 3.12, а, подведено синусоидальное
напряжение частотой f
= 50 Гц с действующим значением U
= 100 В. Параметры элементов схемы: R1
= 30 Ом, L
= 0,1 Гн, C
= 50 мкФ, R2
= 20 Ом. Определить токи в ветвях схемы и
показания приборов. Составить баланс
мощности. Построить в масштабе векторную
диаграмму токов и напряжения.


Рис.
3.12 – Параллельная цепь:
а) схема
замещения; б) векторная диаграмма

Решение

Определяем
комплексные сопротивления параллельных
ветвей. Сопротивление первой ветви

Z1
= R1
+ jXL,

где

XL
= jωL
= 2πfL
= 6,28∙50∙0,1 = 31,4 Ом;

Z1
= 30 + j31,4
Ом.

Комплексное
сопротивление второй ветви

Z2
= R2
jXС;

Z2
= 20 – j63,7
Ом.

Находим комплексные
значения токов в ветвях

I
= I1
+ I2
= 1,6 – j1,64
+ 0,45 + j1,43
= 2,05 – j0,21
A.

Действующие
значения

Для определения
показания вольтметра составляем
уравнение согласно второго закона
Кирхгофа для контура б, в, г, д, б.
Произвольно выбираем направление обходе
контура, показанное на рис. 3.12, а стрелкой

0 = Uбв
+ R2I2
R1I1;

1. . Uбв
= R2I2
R1I1
= 20·(0,45 + j1,43)
– 30(1,6 – j1,64)
=

= 9 + j28,6
– 48 + j49,2
= – 39 + j77,8;

Uбв
= 39 – j77,8
В.

Вольтметр покажет
действующее значение напряжения Uбв

Ваттметр измеряет
мощность, потребляемую активной нагрузкой
(R1
и R2).

Известно, что

Р
= U·I·cosφ.

В этом выражении
неизвестным является cosφ,
где φ
угол сдвига между напряжением U
и током I.
Определить угол φ
(или cosφ)
можно разными путями. Например, cosφ
можно найти из выражения для общего
тока, учитывая, что начальная фаза
напряжения равна нулю. Для этого обратимся
к комплексному значению общего тока

I
= 2,05 – j0,21
A,

где IR
= 2,05 – активная составляющая тока
(проекция комплексного вектора полного
тока на ось действительных чисел);

IX
= – j0,21
– реактивная составляющая тока (проекция
комплексного вектора полного тока на
ось мнимых чисел).

Тогда

где I
= 2,06 А – действующее значение общего
тока.

Показание ваттметра

Р
= 100∙2,06∙0,995 = 205 Вт.

Составим баланс
мощностей.

Полная мощность,
поступающая от источника

где PИ
= 205 Вт; QИ
= 21 Вар.

Мощности приёмников


SП
= РП
+ jQП
= 205 + j21,34
ВА.

Результаты расчётов
показывают, что баланс мощности сходится,
т. е. токи найдены правильно.

Векторную диаграмму
строим на комплексной плоскости, рис.
3.12, б. Выбираем масштабы тока и напряжения:
(Масштаб
выбирается с таким расчётом, чтобы
векторная диаграмма занимала примерно
половину страницы). Откладываем вектор
напряжения совпадающий с осью+1.
Затем откладываем вычисленные значения
токов I1,
I2,
I.
Действительные значения – на оси +1,
мнимые значения – на оси +j.

Контрольные
вопросы к экзамену (зачету)

Контрольные вопросы к зачету
(экзамену ) по разделу “Основы
электротехники”.

1. Электробезопасность.
Характеристики поражения человека
электрическим током.

2. Основные определения: электротехника,
электричество, электрическое поле,
потенциал, напряжение, электрический
ток, источники тока , электродвижущая
сила (ЭДС), закон Ома , законы Кирхгофа.

3. Электрическая цепь. Пассивные и
активные элементы цепи. Параметры
электрической цепи.

4. Расчет электрических
цепей постоянного тока методом законов
Кирхгофа, методом контурных токов.

5. Энергия и мощность
постоянного тока. Баланс мощностей.

6. Переменный ток. Однофазный синусоидальный
ток. Основные параметры: мгновенные,
действующие и средние значения тока,
напряжения и ЭДС. Генерирование
переменного тока.

7. Представление переменного тока
комплексными величинами. Метод комплексных
диаграмм.

8. Метод комплексных амплитуд.
Закон Ома и законы Кирхгофа в комплексной
форме.

9. Активное сопротивление,
индуктивность и емкость в цепи переменного
тока.

10. Последовательная и разветвленные
цепи переменного тока с активным
сопротивлением, емкостью и индуктивностью.
Резонанс тока. Резонанс напряжения.

11.Мощность и энергия в цепи переменного
тока. Активная, реактивная и полная
мощность. Единицы измерения. Баланс
мощностей.

12.Трехфазные электрические цепи. Основные
определения. Линейные и фазные токи и
напряжения. Маркировка фазы. Способы
соединения генераторов и приемников
типа звезда и треугольник. Трехпроводные
и четырехпроводные цепи. Нейтральный
провод.

13. Короткое замыкание фазы. Разрыв
линейного провода. Мощность в цепи
трехфазного тока.

14. Нелинейные электрические цепи.
Аппроксимация нелинейных характеристик.

15. Расчет цепей постоянного тока с одним
или несколькими нелинейными элементами.

16. Основные магнитные величины. Магнитные
цепи постоянного тока.

17. Магнитные цепи переменного тока.
Ферромагнитные материалы.

18. Расчет катушки с магнитопроводом и
воздушным зазором.

19. Энергия и основные потери в
магнитопроводе.

20 Трансформатор. Основные режимы работы.

21. Устройство и принцип действия машин
постоянного тока.

22. Генератор постоянного тока. Основные
характеристики.

23.Двигатель постоянного тока. Основные
характеристики.

24.Устройство и принцип действия машины
переменного тока.

25. Асинхронный двигатель. Основные
характеристики.

26. Синхронный генератор. Основные
характеристики.

Темы
рефератов
.

ЗАДАНИЯ ДЛЯ САМОСТОЯТЕЛЬНОЙ РАБОТЫ

Самостоятельная работа студентов
состоит в изучении ряда теоретических
вопросов по темам дисциплины, перечень
которых приведен в таблице 5 и составления
рефератов..

Таблица 5

п/п

Тема
дисциплины

1

2

1

Тема 1.
Электрические цепи.

Основные
определения, топологические параметры

2

Тема 2. Методы расчета линейных
электрических цепей

3

Тема З. Однофазный переменный ток

4

Тема
4. Электрические цепи трехфазного
тока.

5

Тема 5. Магнитные цепи и электромагнитные
устройства.

6

Тема 6. Трансформаторы

7

Тема 7. Асинхронные машины

8

Тема 8. Машины постоянного тока (МПТ)

9

Тема 9. Синхронные машины

10

Тема 10. Электрические измерения и
приборы

11

Тема 11. Основы электроники и элементной
базы современных электронных устройств

12

Тема 12. Источники вторичного
электропитания

13

Тема 13. Усилители электрических
сигналов

14

Тема 14. Импульсные и автогенераторные
устройства

15

Тема 15. Основы цифровой электроники

16

Тема 16. Микропроцессорные средства

Соседние файлы в предмете [НЕСОРТИРОВАННОЕ]

  • #
  • #
  • #
  • #
  • #
  • #
  • #
  • #
  • #
  • #
  • #

Символический (комплексный) метод расчета цепей переменного тока

Одним из способов расчета цепей переменного тока является комплексный, или еще как говорят, символический метод расчета. Этот метод применяется при анализе схем с гармоническими ЭДС, напряжениями и токами. В результате решения получают комплексное значение токов и напряжений, используя для решения любые методы (эквивалентных преобразований, контурных токов, узловых потенциалов и т.п.). Но для начала необходимо иметь понятие, в каких именно формах может представляться синусоидальная величина. 1. Одна из форм представления – это вращающийся вектор (см. рис.1):

Рис.1. Вращающийся вектор

С помощью рисунка ясно видно, как с течением времени меняется значение синусоидальной величины. В нашем случае – это величина а на графике, которая может быть, например, входным напряжением. Величина имеет некоторое начальное значение при t = 0 при начальной фазе φ

имеет положительное максимальное значение при угле ωt3, когда при времени t3 сумма ωt3 + φ = 90° и соответственно,

имеет отрицательное максимальное значение при угле ωt7, когда при времени t7 сумма углов ωt7 + φ = 270° и, соответственно,

и имеет два нулевых значения при ωtn + φ = 0, когда ωtn = —φ (на рис.1 эта область не показана и находится слева от начала координат)

и имеет нулевое значение при угле ωt11, когда при времени t11 сумма ωt11 + φ = 360° и соответственно,

Именно по такому закону и меняется привычное нам переменное напряжение 220 В, изменяясь по синусоидальному закону от значения 0 В до максимальных 311 В и обратно.

2. Другая форма представления – это комплексное число. Чтобы представить ранее рассмотренную форму представления синусоидальной величины, которая имеет некоторую начальную фазу φ, создают комплексную плоскость в виде графика зависимости двух величин (рис.2)

Рис.2. Комплексное число на комплексной плоскости

Длина вектора Am на такой комплексной плоскости равна амплитуде (максимальному значению) рассматриваемой величины. С учетом начальной фазы φ такое число записывают как .

На практике при использовании для расчетов символического (комплексного) метода расчета используют для некоторых удобств не амплитудное значение величины, а так называемое действующее значение. Его величина в корень из двух раз меньше амплитудного и обозначается без индекса m, т.е. равна

На рисунке выше этот вектор также показан.
Например, при том же нашем напряжении в сети, максимальное значение синусоидально изменяющегося напряжения равно 311 В, а действующее значение, к значению которого мы привыкли

При работе с комплексными числами и расчетов применяют различные формы записи комплексного числа. Например, при сложении комплексных чисел удобнее использовать алгебраическую форму записи таких чисел, а при умножении или делении – показательную форму записи. В некоторых случаях пишут тригонометрическую форму.
Итак, три формы записи комплексного числа:

1) показательная форма в виде

2) тригонометрическая форма в виде

3) алгебраическая форма

где ReA — это действительная составляющая комплексного числа, ImA — мнимая составляющая.

Например, имеем комплексное число в показательной форме вида

в тригонометрической форме записи это запишется как

при подсчете получим число, плавно переходящее в алгебраическую форму с учетом того, что

В итоге получим

При переходе от алгебраической формы к показательной комплексное число вида

переходит к показательному виду по следующим преобразованиям

Таким образом, и получим

Перейдем к рассмотрению несложных примеров использования символического, или по-другому, комплексного метода расчета электрических цепей. Составим небольшой алгоритм комплексного метода:

      • Составить комплексную схему, заменяя мгновенные значения ЭДС, напряжений и токов их комплексным видом
      • В полученной схеме произвольно выбирают направления токов в ветвях и обозначают их на схеме.
      • При необходимости составляют комплексные уравнения по выбранному методу решения.
      • Решают уравнения относительно комплексного значения искомой величины.
      • Если требуется, записывают мгновенные значения найденных комплексных величин.

Пример 1. В схеме рис.3 закон изменения ЭДС e = 141sin*ωt. Сопротивления R1 = 3 Ом, R2 = 2 Ом, L = 38,22 мГн, С = 1061,6 мкФ. Частота f = 50 Гц. Решить символическим методом. Найти ток и напряжения на элементах. Проверить 2-ой закон Кирхгофа для цепи.

Рис.3. Схема с последовательным соединением элементов

Составляем комплексную схему, обозначив комплексные токи и напряжения (рис.4):

Рис.4. Схема с комплексными обозначениями

По закону Ома ток в цепи равен

где U — комплексное входное напряжение, Z — полное сопротивление всей цепи. Комплекс входного напряжения находим как

Пояснение: здесь начальная фаза φ = 0°, так как общее выражение для мгновенного значения напряжение вида при φ = 0° равно

Соответственно, комплекс входного напряжения в показательной форме запишется как

Полное комплексное сопротивление цепи в общем виде

Находим комплексное сопротивление индуктивности

Находим комплексное сопротивление емкости

Соответственно, общее комплексное сопротивление цепи

Комплексные напряжения на элементах

Проверяем второй закон Кирхгофа для замкнутого контура, т.е. должно выполняться равенство

С небольшим расхождением из-за округлений промежуточных вычислений всё верно.

Пример 2. В электрической цепи (рис.5) однофазного синусоидального тока, схема и параметры элементов которой заданы для каждого варианта в таблице, определить:
1) полное сопротивление электрической цепи и его характер;
2) действующие значения токов в ветвях;
3) показания вольтметра и ваттметра;

      Исходные данные: Е = 220 В, f = 50 Гц, L1 = 38,2 мГн, R2 = 6 Ом, С2 = 318 мкФ, L2 = 47,7 мГн, R3 = 10 Ом, С3 = 300 мкФ.

Рис.5.Цепь однофвзного синусоидального тока

Решение:
1. Находим комплексные сопротивления ветвей и всей цепи:
Учитываем, что

Комплексное сопротивление первой ветви:

Комплексное сопротивление второй ветви:

Комплексное сопротивление третьей ветви:

Общее сопротивление цепи

— нагрузка носит активно-индуктивный характер

2. Находим действующие значения токов в ветвях:

Рис.6. Схема с обозначенными комплексными токами

Действующие значения, соответственно,

3. Определим показания приборов:
Вольтметр подключен по схеме параллельно источнику питания. Соответственно его показание равно:
U=220 В
Ваттметр включен токовой обмоткой в разрыв третьей ветви, а обмоткой напряжения также к выводам третьей ветви, измеряя, таким образом, активную мощность третьей ветви. Эта мощность равна мощности на сопротивлении R3. Его показания:

Основы символического метода расчета. Методы контурных токов и узловых потенциалов.

Закон Ома для участка цепи с источником ЭДС

Возьмем два участка цепи a – b и c – d (см. рис. 1) и составим для них уравнения в комплексной форме с учетом указанных на рис. 1 положительных направлений напряжений и токов.

Объединяя оба случая, получим

или для постоянного тока

Формулы (1) и (2) являются аналитическим выражением закона Ома для участка цепи с источником ЭДС , согласно которому ток на участке цепи с источником ЭДС равен алгебраической сумме напряжения на зажимах участка цепи и ЭДС, деленной на сопротивление участка. В случае переменного тока все указанные величины суть комплексы. При этом ЭДС и напряжение берут со знаком “+”, если их направление совпадает с выбранным направлением тока, и со знаком “-”, если их направление противоположно направлению тока.

Основы символического метода расчета цепей
синусоидального тока

Расчет цепей переменного синусоидального тока может производиться не только путем построения векторных диаграмм, но и аналитически – путем операций с комплексами, символически изображающими синусоидальные ЭДС, напряжения и токи. Достоинством векторных диаграмм является их наглядность, недостатком – малая точность графических построений. Применение символического метода позволяет производить расчеты цепей с большой степенью точности.

Символический метод расчета цепей синусоидального тока основан на законах Кирхгофа и законе Ома в комплексной форме.

Уравнения, выражающие законы Кирхгофа в комплексной форме, имеют совершенно такой же вид, как и соответствующие уравнения для цепей постоянного тока. Только токи, ЭДС, напряжения и сопротивления входят в уравнение в виде комплексных величин.

1. Первый закон Кирхгофа в комплексной форме:

2. Второй закон Кирхгофа в комплексной форме:

или применительно к схемам замещения с источниками ЭДС

3. Соответственно матричная запись законов Кирхгофа в комплексной форме имеет вид:

§ первый закон Кирхгофа:

§ второй закон Кирхгофа

Определить: 1) полное комплексное сопротивление цепи ;
2) токи
Рис. 2

1. .

2. .

3.

.

4. Принимая начальную фазу напряжения за нуль, запишем:

.

.

5. Поскольку ток распределяется обратно пропорционально сопротивлению ветвей (это вытекает из закона Ома), то

6. .

7. Аналогичный результат можно получить, составив для данной схемы уравнения по законам Кирхгофа в комплексной форме

или после подстановки численных значений параметров схемы

Специальные методы расчета

Режим работы любой цепи полностью характеризуется уравнениями, составленными на основании законов Кирхгофа. При этом необходимо составить и решить систему с n неизвестными, что может оказаться весьма трудоемкой задачей при большом числе n ветвей схемы. Однако, число уравнений, подлежащих решению, может быть сокращено, если воспользоваться специальными методами расчета , к которым относятся методы контурных токов и узловых потенциалов.

Метод контурных токов

Идея метода контурных токов: уравнения составляются только по второму закону Кирхгофа, но не для действительных, а для воображаемых токов, циркулирующих по замкнутым контурам, т.е. в случае выбора главных контуров равных токам ветвей связи. Число уравнений равно числу независимых контуров, т.е. числу ветвей связи графа . Первый закон Кирхгофа выполняется автоматически. Контуры можно выбирать произвольно, лишь бы их число было равно и чтобы каждый новый контур содержал хотя бы одну ветвь, не входящую в предыдущие. Такие контуры называются независимыми . Их выбор облегчает использование топологических понятий дерева и ветвей связи.

Направления истинных и контурных токов выбираются произвольно. Выбор положительных направлений перед началом расчета может не определять действительные направления токов в цепи. Если в результате расчета какой-либо из токов, как и при использовании уравнений по законам Кирхгофа, получится со знаком “-”, это означает, что его истинное направление противоположно.

Пусть имеем схему по рис. 3.

Выразим токи ветвей через контурные токи:

;

; ;

; .

Обойдя контур aeda, по второму закону Кирхгофа имеем

.

Поскольку ,

.

Таким образом, получили уравнение для первого контура относительно контурных токов. Аналогично можно составить уравнения для второго, третьего и четвертого контуров:

совместно с первым решить их относительно контурных токов и затем по уравнениям, связывающим контурные токи и токи ветвей, найти последние.

Однако данная система уравнений может быть составлена формальным путем:

При составлении уравнений необходимо помнить следующее:

– сумма сопротивлений, входящих в i- й контур;

– сумма сопротивлений, общих для i- го и k- го контуров, причем ;

члены на главной диагонали всегда пишутся со знаком “+”;

знак “+” перед остальными членами ставится в случае, если через общее сопротивление i- й и k- й контурные токи проходят в одном направлении, в противном случае ставится знак “-”;

если i- й и k- й контуры не имеют общих сопротивлений, то ;

в правой части уравнений записывается алгебраическая сумма ЭДС, входящих в контур: со знаком “+”, если направление ЭДС совпадает с выбранным направлением контурного тока, и “-”, если не совпадает.

В нашем случае, для первого уравнения системы, имеем:

Следует обратить внимание на то, что, поскольку , коэффициенты контурных уравнений всегда симметричны относительно главной диагонали.

Если в цепи содержатся помимо источников ЭДС источники тока, то они учитываются в левых частях уравнений как известные контурные токи: k- й контурный ток, проходящий через ветвь с k- м источником тока равен этому току .

Метод узловых потенциалов

Данный метод вытекает из первого закона Кирхгофа. В качестве неизвестных принимаются потенциалы узлов, по найденным значениям которых с помощью закона Ома для участка цепи с источником ЭДС затем находят токи в ветвях. Поскольку потенциал – величина относительная, потенциал одного из узлов (любого) принимается равным нулю. Таким образом, число неизвестных потенциалов, а следовательно, и число уравнений равно , т.е. числу ветвей дерева .

Пусть имеем схему по рис. 4, в которой примем .

Допустим, что и известны. Тогда значения токов на основании закона Ома для участка цепи с источником ЭДС

Запишем уравнение по первому закону Кирхгофа для узла а :

и подставим значения входящих в него токов, определенных выше:

.

Сгруппировав соответствующие члены, получим:

.

Аналогично можно записать для узла b :

.

Как и по методу контурных токов, система уравнений по методу узловых потенциалов может быть составлена формальным путем. При этом необходимо руководствоваться следующими правилами:

1. В левой части i- го уравнения записывается со знаком “+”потенциал i- го узла, для которого составляется данное i- е уравнение, умноженный на сумму проводимостей ветвей, присоединенных к данному i- му узлу, и со знаком “-”потенциал соседних узлов, каждый из которых умножен на сумму проводимостей ветвей, присоединенных к i- му и k- му узлам.

Из сказанного следует, что все члены , стоящие на главной диагонали в левой части системы уравнений, записываются со знаком “+”, а все остальные – со знаком “-”, причем . Последнее равенство по аналогии с методом контурных токов обеспечивает симметрию коэффициентов уравнений относительно главной диагонали.

2. В правой части i- го уравнения записывается так называемый узловой ток , равный сумме произведений ЭДС ветвей, подходящих к i- му узлу, и проводимостей этих ветвей. При этом член суммы записывается со знаком “+”, если соответствующая ЭДС направлена к i- му узлу, в противном случае ставится знак “-”. Если в подходящих к i- му узлу ветвях содержатся источники тока, то знаки токов источников токов, входящих в узловой ток простыми слагаемыми, определяются аналогично.

В заключение отметим, что выбор того или иного из рассмотренных методов определяется тем, что следует найти, а также тем, какой из них обеспечивает меньший порядок системы уравнений. При расчете токов при одинаковом числе уравнений предпочтительнее использовать метод контурных токов, так как он не требует дополнительных вычислений с использованием закона Ома. Метод узловых потенциалов очень удобен при расчетах многофазных цепей, но не удобен при расчете цепей со взаимной индуктивностью.

1. Основы теории цепей: Учеб.для вузов /Г.В.Зевеке, П.А.Ионкин, А.В.Нетушил, С.В.Страхов. –5-е изд., перераб. –М.: Энергоатомиздат, 1989. -528с.

2. Бессонов Л.А. Теоретические основы электротехники: Электрические цепи. Учеб. для студентов электротехнических, энергетических и приборостроительных специальностей вузов. –7-е изд., перераб. и доп. –М.: Высш. шк., 1978. –528с.

Контрольные вопросы и задачи

1. В ветви на рис. 1 . Определить ток .

Ответ: .

2. В чем заключается сущность символического метода расчета цепей синусоидального тока?

3. В чем состоит сущность метода контурных токов?

4. В чем состоит сущность метода узловых потенциалов?

5. В цепи на рис. 5 ; ; ; . Методом контурных токов определить комплексы действующих значений токов ветвей.

Ответ: ; ; .

6. В цепи на рис. 6 . Рассчитать токи в ветвях, используя метод узловых потенциалов.

Ответ: ; ; ; ; ; ; .

Символический метод расчета цепей

Содержание:

Символический метод расчета цепей:

Символический метол, введенный в теорию переменных токов Штейнмецом, является аналитическим развитием векторных диаграмм. Он основан на изображении векторов в комплексной плоскости и на их записи комплексными числами. Это приводит к применению для цепей синусоидального переменного тока законов Ома и Кирхгофа и вытекающих из них методов расчета цепей в той же форме, что и для цепей постоянного тока. В России символический метод был введен В. Ф. Миткевичем.

В символическом методе принято исходную ось направлять вертикально и на ней откладывать вверх положительные вещественные числа, а по горизонтальной оси влево — положительные мнимые числа (рис. 8.1). В дальнейшем эти оси называются осью и осью мнимых. Тогда, например, вращающийся вектор Um, изображающий синусоидальное напряжение

и составляющий с осью вещественных угол может быть записан в виде комплексного числа в алгебраической, тригонометрической или показательной форме:

здесь — составляющие, соответственно, по осям вещественных и мнимых, Um — модуль (величина) вектора, угол — его аргумент, а е — основание натуральных логарифмов.

Комплекс называют множителем вращения, а — комплексной амплитудой. Соответственно

называют комплексным действующим значением, в данном примере — напряжения, или комплексным напряжением. На комплексной плоскости оно изображается неподвижным вектором.

Для обратного перехода от комплекса к мгновенному значению и следует взять только мнимую часть комплекса (без i), что записывается следующим образом:

Таким образом, комплекс является также изображением (как бы символом) синусоиды и, откуда и получил свое название метод, заключающийся в замене оригиналов (синусоид) комплектными изображениями, в операциях над ними и затем в обратном переходе для искомых величин от их изображений к оригиналам.

Геометрическому сложению и вычитанию векторов соответствует алгебраическое сложение и вычитание их проекций на оси комплексной плоскости, т. е. их вещественных и мнимых составляющих. Поэтому геометрическое сложение и вычитание векторов должно быть заменено вновь алгебраическим сложением и вычитанием их комплексов. Таким образом, алгебраический характер сложения и вычитания мгновенных значений синусоидальных величин сохраняется при замене оригиналов комплексными изображениями.

Так как проекция произведения двух векторов не равна произведению проекций этих векторов, изображение произведения двух синусоидальных функций не равно произведению их изображений, поэтому прч умножении таких функций нельзя применять символический метод.

Производная синусоидальной функции

так как Полученное изображение равно производной изображения исходной функции:

Интеграл той же синусоидальной функции

равное интегралу изображения исходной функции:

Таким образом, однозначное соответствие имеет место также между производными и интегралами оригинала и комплексного изображения.

Здесь получен еще один важный результат: дифференцированию оригинала соответствует, умножение на его изображения, интегрированию — деление на . Следовательно, интегро-дифференциальному уравнению для мгновенных значений соответствует алгебраическое уравнение для изображений, т. е. применение символического метода приводит к алгебраизации этих уравнений, что крайне упрощает расчеты.

Применение символического метода для расчета цепей переменного тока

Применение символического метода можно показать на примере. Так, для цепи с последовательным соединением r, L и С уравнению по второму закону Кирхгофа

при синусоидальном законе изменения напряжения и тока соответствует алгебраическое уравнение

(8.1)

откуда комплексное изображение тока

(8 2)

От изображения можно сделать переход к оригиналу — мгновенному значению тока.

Выражение (8.2) можно рассматривать как закон Ома в символической форме. Тогда знаменатель

может рассматриваться как комплексное полное сопротивление. Его модуль z равен полному сопротивлению цепи, его аргумент — сдвигу фаз между напряжением и током цепи. Графически Z изображается неподвижным вектором с составляющими — активным сопротивлением r по оси вещественных и реактивным х — по оси мнимых, что показано на рис. 8.2 для случая > 0. Соответствующий прямоугольный треугольник является треугольником сопротивлений.

Необходимо заметить, что знак плюс, стоящий в общем выражении комплексного сопротивления Z =г + jx, сохраняется в конкретном числовом выражении при преобладании индуктивного сопротивления ( > 0) и переходит в минус при преобладании емкостного сопротивления ( 0. Вектор У имеет направление, сопряженное с направлением обратного ему вектора Z. Знак минус, стоящий в общем выражении комплекса проводимости Y = g — jb, сохраняется в конкретном числовом выражений при >0 и переходит в плюс при

Действительные Мгновенные Действующие Комплексные
Внешние Внешние и внутренние Внешние >
Омические Активные Полные Комплексные
Алгебраические > Геометрические Алгебраические

Непосредственное применение символического метода к вычислению по напряжению и току мощности, мгновенное значение которой является произведением их мгновенных значений (р = ui), невозможно. Однако для вычисления активной, реактивной и полной мощности по символическим изображениям напряжения и тока может быть использован искусственный прием. Для этого комплексное напряжение должно быть умножено на комплекс I, сопряженный с комплексным током

Таким образом, вещественная часть комплексной мощности S равна активной мощности Р, а мнимая — реактивной Q. При этом положительный знак сохраняется для индуктивной мощности и изменяется на отрицательный для емкостной. Полная мощность вычисляется, как модуль комплексной мощности:

Расчет цепей переменного тока символическим методом

При расчете цепей по законам Кирхгофа методика составления уравнений остается той же, что и при постоянном токе. Для заданных комплексных э. д. с. и токов должны быть также указаны их положительные направления, для искомых — ими надо задаться.

Например, для цени рис. 7.21, а с двумя узлами и двумя элементарными контурами по первому закону Кирхгофа должно быть составлено одно уравнение

Два уравнения, составляемые по второму закону Кирхгофа, при обходе элементарных контуров А и В по часовой стрелке, будут

При постоянном токе ответ со знаком минус указывал на встречное направление по сравнению с предположенным, а при переменном токе ответ в виде комплекса является окончательным для принятого направления искомой величины — напряжения или тока. При выборе обратного направления фаза (аргумент) искомого комплекса изменилась бы на угол π.

Аналогичным образом составляются и решаются уравнения при применении остальных методов, вытекающих из законов Кирхгофа. Так, уравнения по методу контурных токов для цепи рис. 7.21, а при обходе контуров A и В по часовой стрелке имеют вид:

где

Символический метод весьма удобен также для решения задач в общем виде.

В электроизмерительной технике широко применяется мост переменного тока (рис. 8.3). Условие равновесия моста постоянного тока имеет вид:

По аналогии условие равновесия моста переменного тока:

Это условие распадается на два — равенство модулей и аргументов левой и правой частей:

Если модули и аргументы полных сопротивлений трех ветвей известны, из этих уравнений могут быть определены модуль и аргумент полного сопротивления четвертой ветви.

Вторым примером применения символического метода для решения задач в общем виде может служить задача поддержания в цепи изменяющейся нагрузки неизменного по величине и фазе тока. Например, при последовательном соединении ламп, применяемом при освещении аэродромов, должны автоматически замыкаться накоротко зажимы перегоревшей лампы, чтобы избежать разрыва цепи при этом ток остальных не должен измениться.

Пусть для схемы рис. 8.4, а, питаемой напряжением U = const, требуется найти условие, при выполнении которого ток I в правой параллельной ветви не будет меняться по величине и по фазе при любом изменении сопротивления Z этой ветви.

Общее выражение для комплекса тока I может быть найдено методом эквивалентного источника напряжения. По аналогии с цепью постоянного тока

Здесь комплекс напряжения между зажимами разомкнутой ветви Z (рис. 8.4, б) и комплекс полного сопротивления ZB цепи относительно зажимов ветви Z при источнике напряжения, замкнутом накоротко (рис. 8.4, в), соответственно равны:
а искомый ток

Для того чтобы ток I не зависел от сопротивления Z нагрузки, коэффициент при Z в выражении I должен быть равен нулю:

Это будет выполнено, если

т. е. сопротивления Z1 и Z2 должны быть чисто реактивными, равными
по величине и противоположными по знаку. Одно из них будет индуктивным, а другое — емкостным:

При этом ток нагрузки

Если в цепь до разветвления включено индуктивное сопротивление, а потом — емкостное (рис. 8.5, а), то ток

отстает по фазе от приложенного к цепи напряжения на угол π2. Если индуктивное и емкостное сопротивления поменять местами (рис. 8.5, б), то

  1. т. е. ток I опережает приложенное к цепи напряжение на угол π/2. При изменении Z ток I1 до разветвления изменяется и по величине

и по фазе от значения (резонанс напряжений).

Метод дуальных цепей

Метод дуальных цепей, рассмотренный в для частного случая резонансных цепей, является общим методом. Взаимная замена величин при их символической записи должна осуществляться по табл. 8.2, вытекающей из табл. 7.1.

Таблица 8.2

Последовательное соединение Параллельное соединение ω U I L C r g Z Y
Параллельное соединение Последовательное соединение ω I U C L g r Y Z

Отсюда можно получить соотношения для дуальной цепи, если они даны для цепи исходной. Так, если для исходной цепи в какой-либо вегви имеет место короткое замыкание (Z = 0), то в дуальной цепи это соответствует холостому ходу (У = 0), и наоборот. При переходе от исходной цепи к дуальной уравнения по первому и второму законам Кирхгофа меняются местами.

Основным свойством дуальных цепей является неизменность их параметров r, L и С при переменной частоте. Например, в дуальных цепях рис. 8.6, а и б численное равенство сопротивления и проводимости сохраняется при изменении частоты. Этим дуальные цепи отличаются от эквивалентных последовательных и параллельных схем, в которых при изменении частоты и постоянстве параметров одной схемы параметры другой изменяются.

Это свойство дуальных цепей позволяет, произведя исследование поведения какой-либо цепи при переменной частоте, перенести результаты на дуальную цепь, заменив напряжения токами и т. д., что и было сделано для резонансных цепей.

При переходе к дуальной цепи не изменяют своей величины мощности S, Р и Q, так как в их выражения входят произведение напряження и тока, и лишь у реактивной мощности Q = VI sin изменяется знак: индуктивная мощность заменяется емкостной, и наоборот.

В качестве примера может быть решена задача создания схем преобразования неизменного по величине и фазе тока в неизменное по величине и фазе напряжение, т. е. схем, дуальных со схемами. При замене схем и величин по табл. 8.2 получается схема рис. 8.7, а, дуальная схеме рис. 8.5, а, и схема рис. 8.7, б, дуальная схеме рис. 8.5, б. Если

то при неизменном токе I напряжение О на изменяющейся проводимости Y будет постоянным, т. е.

что получается путем перехода от формул для токов I исходных цепей.

Символический метод электрических цепей переменного тока

Методы расчета электрических цепей переменного тока при помощи векторных диаграмм, рассмотренные в предыдущих главах, основаны на изображении синусоидальных величин векторами.

Из курса математики известно, что каждому вектору А в комплексной плоскости (рис. 15.1) соответствует комплексное число А, которое можно выразить в форме:
алгебраической —


Рис. 15.1. К вопросу о выражении вектора комплексным числом

тригонометрической —

показательной —
Это дает основание от графического (векторного) выражения синусоидальных напряжений и токов перейти к аналитическому выражению их комплексными числами, а операции с векторами заменить алгебраическими действиями.

Выражение характеристик электрических цепей комплексными числами

При расчете электрических цепей переменного тока используют или определяют следующие величины: э.д.с. напряжения, токи, сопротивления и проводимости, мощность. Все эти величины должны быть выражены в символической форме, т. е. комплексными числами.

Напряжения и токи

Подобно тому как на векторных диаграммах длины векторов выражают действующие величины, комплексные выражения э. д. с. .напряжений и токов записывают так, что модули их также равны действующим величинам (комплексы синусоидально изменяющихся величин принято отмечать точками над их буквенными обозначениями (например, комплексы напряжения тока ). Комплексы величин, не зависящих от времени (например, сопротивлений, проводимостей), обозначают большими буквами без точек, но с черточкой внизу: )

Для примера рассмотрим схему электрической цепи параллельного соединения катушки и конденсатора (рис. 15.2).
Напряжение на зажимах цепи выражается уравнением

Этому напряжению соответствуют вектор U в комплексной плоскости (рис. 15.3) и комплексное число в показательной форме

Ток i1 в катушке отстает от напряжения на угол φ1:

угол в рассматриваемом случае

Вектору тока I1 соответствует комплексное число

Ток в конденсаторе опережает напряжение на угол φ2. Вектору тока I2 соответствуют уравнение

и комплекс

где

Согласно первому закону Кирхгофа, ток в неразветвленной части цепи складывается из токов в параллельных ветвях:

Для определения этого тока сложение векторов I1 и I2 можно заменить сложением комплексов:



Следует обратить внимание на различие между действительной или мнимой частями комплекса, с одной стороны, и активной или реактивной составляющими вектора тока — с другой.
Действительная и мнимая части комплекса тока равны проекциям вектора тока на оси комплексной плоскости (ось действительных и ось мнимых величин).

Активная и реактивная составляющие вектора тока в данном участке цепи равны его проекциям на взаимно перпендикулярные оси, одна из которых направлена вдоль вектора напряжения этого же участка цепи. Действительная и мнимая части комплекса тока равны соответственно активной и реактивной составляющим вектора тока только в том случае, если вектор напряжения направлен вдоль оси действительных чисел, т. е. комплекс напряжения выражается действительным числом.

Рис. 15.2. К вопросу о выражении токов, напряжений, сопротивлений проводимостей комплексными числами

Рис. 15.3. Векторная диаграмма к схеме цепи рис. 15.2

Сопротивления

Для выражения сопротивлений в комплексной форме продолжим рассмотрение схемы рис. 15.2, где каждый из элементов (катушка и конденсатор) представлен активным и реактивным сопротивлениями, соединенными последовательно.

Разделив комплекс напряжения на комплекс тока в катушке , получим комплекс сопротивления первой ветви:

где — модуль комплекса полного сопротивления; — угол сдвига фаз между напряжением и током первой ветви .
Выразим комплекс сопротивления катушки в тригонометрической и алгебраической форме:

Но , поэтому

Аналогично, для второй ветви

где —модуль комплекса полного сопротивления; — угол сдвига фаз между напряжением и током второй ветви

или

Если в ветвях схемы рис. 15.2 реактивных сопротивлений нет то, согласно выражениям (15.6) и (15.7), При

Из приведенных рассуждений следует:

  1. Активное сопротивление в комплексной форме выражается действительным положительным числом.
  2. Реактивные сопротивления в комплексной форме выражаются мнимыми числами, причем индуктивное сопротивление (ХL) положительно, а емкостное (ХC) отрицательно.
  3. Полное сопротивление участка цепи при последовательном соединении R и X выражается комплексным числом, действительная часть которого равна активному сопротивлению, а мнимая часть равна реактивному сопротивлению этого участка.

Проводимости

Выражения проводимостей ветвей в комплексной форме можно получить, представив каждый элемент (катушку и конденсатор) схемой параллельного соединения активной и реактивной проводимостей (см. рис. 14.1, б)

Из этих формул видно, что выражения проводимостей комплексными числами можно получить в таком же порядке, как для сопротивлений. Для того чтобы не повторять аналогичных рассуждений, полные проводимости в символической форме можно найти как величины, обратные комплексам полных сопротивлений:

Для первой ветви (катушки)



где и — активная и индуктивная проводимости.
Для второй ветви (конденсатора)


где и — активная и емкостная проводимости.
Результаты этих преобразований показывают, что полная проводимость ветви электрической цепи в комплексной форме выражается комплексным числом, действительная часть которого равна активной проводимости, а мнимая часть равна реактивной проводимости этой ветви, причем индуктивная проводимость отрицательна, а емкостная — положительна.

Мощность

Комплекс мощности в данной цепи определяется умножением комплекса напряжения на сопряженный комплекс тока этой цепи.
Для ветви с активным сопротивлением и индуктивностью (см. рис. 15.2), согласно векторной диаграмме (см. рис. 15.3),

Произведение комплекса напряжения и сопряженного комплекса тока

В алгебраической форме

Действительная часть полученного комплекса выражает активную мощность, а мнимая часть без множителя — реактивную мощность первой ветви.
Для ветви с активным сопротивлением и емкостью


В алгебраической форме

Реактивная мощность в цепи с емкостью имеет отрицательный знак в отличие от положительного знака реактивной мощности в цепи с индуктивностью. Модуль комплекса мощности в той и другой ветви равен полной мощности:


Рис. 15.4. К вопросу о преобразовании схем с применением комплексных чисел

Основные уравнения электрических цепей в комплексной форме

Представление векторов напряжений и токов комплексами, выражение сопротивлений и проводимостей комплексными числами, а также замена операций с векторами алгебраическими действиями с комплексными числами позволяют значительно упростить расчет сложных цепей переменного тока. Кроме того, применение комплексных чисел обеспечивает единство методов расчета электрических цепей постоянного и переменного токов. Это значит, что все методы расчета и вытекающие из них соотношения для цепей постоянного тока можно применить и для цепей переменного тока, если величины выражены в комплексной форме. В этом практический смысл применения комплексных чисел для решения задач электротехники.

Законы Кирхгофа

Согласно первому закону Кирхгофа, алгебраическая сумма комплексов токов в электрическом узле равна нулю:

Для составления уравнения в символической форме по первому закону Кирхгофа нужно выбрать условно-положительные направления токов. В уравнении (15.15) ток записывают со знаком плюс, если он направлен к узлу. Для схемы рис. 14.15, а

или

а в комплексной форме

или

Согласно второму закону Кирхгофа, в контуре электрической цепи алгебраическая сумма комплексов э. д. с. источников равна алгебраической сумме комплексов падений напряжения:

Для схемы рис. 14.10

а в комплексной форме

Преобразование схем

На примере цепи смешанного соединения сопротивлений (рис. 15.4) рассмотрим расчет методом преобразования и упрощения схемы. Параллельно соединенные ветви, имеющие полные сопротивления

заменяются одной ветвью с эквивалентным сопротивлением

Сопротивление в неразветвленной части цепи соединено последовательно с сопротивлением

Общее сопротивление цепи

Ток в неразветвленной части цепи

Напряжения на участках, цепи:

Токи в параллельных ветвях:

Преобразованием можно упростить и более сложные схемы с последовательным и параллельным соединениями участков, а также схемы, которые содержат треугольники или трехлучевые звезды сопротивлений.

Метод узлового напряжения

Схему с двумя узлами можно рассчитать, определив узловое напряжение по формуле

Эта формула аналогична формуле (4.21). В числителе ее записана алгебраическая сумма произведений комплексов э. д. с. и проводимости всех ветвей, а в знаменателе — сумма комплексов проводимостей ветвей.
Комплекс тока определяют по формуле

Правило выбора знаков э.д. с. в формулах (15.16) — (15.18) такое же, как и в цепи постоянного тока, с той лишь разницей, что условно-положительные направления э. д. с. выбираются при расчете, а в цепи постоянного тока направления э. д. с. обычно заданы.

Метод эквивалентного генератора

Порядок расчета по методу эквивалентного генератора, для цепей постоянного тока, пригоден и для цепей переменного тока, если э.д. с., токи и сопротивления их выражены в комплексной форме.
Ток в исследуемой ветви определяют из уравнения, подобного (5.12):

где — комплекс эквивалентной э.д.с., равный комплексу напряжения холостого хода активного двухполюсника при отключении исследуемой ветви, — комплекс сопротивления пассивного двухполюсника относительно точек присоединения исследуемой ветви (комплекс внутреннего сопротивления эквивалентного генератора); — комплекс сопротивления исследуемой ветви.

Задача 15.3.

Выполнить символическим методом расчет цепи (см. рис. 14.8). Дано:



= 8 Ом; Х21 = 6 Ом; Х1С — 15 Ом; Х2С = 10 Ом.


Определить ток в цепи и напряжения
Решение. Выразим заданные э. д. с. и сопротивления комплексными числами.
Э. д. с. в комплексной форме:


Сопротивления в комплексной форме:

При последовательном соединении общее сопротивление цепи


Сопротивление цепи в показательной форме:

модуль

аргумент

Угол φ можно определить, найдя


Ток в цепи

Для удобства деления выразим числитель и знаменатель в показательной форме:


Из сравнения комплексов и и обшей з. д. с. видно, что ток в цепи совпадает по фазе с э. д. с. Е2 и опережает общее значение э. д. с. на угол 120—83 = 37°.

Напряжение



Угол сдвига фаз между током и напряжением

Напряжение

Между током и напряжением угол сдвига фаз
так как

Задача 15.5.

Определить символическим методом напряжения ка зажимах источника, токи и мощность в цепи рис. 14.13, для которой известны R1 = 8 Ом; ХL = 6 Ом; R2 = 9 Ом; ХC = 12 Ом; I1 = 9А.
Решение. Выразим сопротивления ветвей в символической форме:


Предположим, что комплекс тока выражается действительным числом (начальная фаза тока )

(начальную фазу тока можно выбрать произвольно, т.е. угол не равен нулю).
Напряжение в первой ветви, равное напряжению на зажимах источника,

Ток во второй ветви

Ток в источнике


Мощность цепи

Комплексные сопротивления и проводимости элементов электрических цепей

Вычисление комплексных сопротивлений и проводимостей последовательных и параллельных двухполюсников, содержащих различные элементы электрических цепей, осуществляются по тем же правилам, которые были получены для резистивных цепей, поскольку, как это было показано в лекции 7, для комплексных амплитуд справедливы законы Ома и Кирхгофа.

Комплексные сопротивления и проводимости полностью характеризуют свойства соответствующего элемента. Будем рассматривать только пассивные элементы, через которые проходит гармонический ток

(8.1)

комплексная амплитуда которого равна Найдём комплексные сопротивления и проводимости резистивного элемента, индуктивности и ёмкости при согласованной системе отсчёта токов и напряжений.

Резистивный элемент

Для резистивного элемента, обладающего активным сопротивлением, имеем

где — амплитуда гармонического напряжения. Отсюда комплексная амплитуда напряжения на резистивном элементе

По определению комплексного сопротивления двухполюсника (7.38) имеем:

(8.3)

а комплексная проводимость

Средняя мощность, выделяемая в активном сопротивлении, согласно (7.15) при равна

(8.4)

или, переходя к действующим значениям (7.18) напряжения и тока,

(8.5)

Выводы:

Индуктивность

Напряжение на зажимах индуктивности изменяется по закону

(8.6)

Операции дифференцирования гармонического колебания (см. лекцию 7) соответствует умножение символического изображения на оператор т. е.

(8.7)

причём зависимость между амплитудами гармонических колебаний напряжения на зажимах индуктивности и тока в индуктивности определяется выражением:

(8.8)

Из (8.7) для индуктивности получаем: комплексное сопротивление (индуктивное сопротивление)

(8.9)

и комплексную проводимость (индуктивную проводимость)

(8.10)

Выводы:

Комплексные сопротивление (8.9) и проводимость (8.10) индуктивности имеют только реактивные составляющие и зависят от частоты:

поэтому элемент индуктивности называют реактивным;

гармоническое напряжение на индуктивности опережает ток на поскольку

что следует из (8.6), т. е. ток и напряжение находятся в квадратуре (рис. 8.1, б);

значение средней мощности в элементе индуктивности равно нулю:

это объясняется тем, что в элементе индуктивности энергия не рассеивается; в режиме гармонических колебаний происходит обмен энергией между индуктивностью и подключённой к ней внешней цепью.

Ёмкость

Напряжение на зажимах ёмкости определяется соотношением

(8.11)

Операции интегрирования гармонического колебания (см. лекцию 7) соответствует деление символического изображения на оператору’со, т. е.

(8.12)

причём зависимость между амплитудами гармонических колебаний напряжения на зажимах ёмкости и тока в ёмкости определяется выражением:

(8.13)

Из (8.12) для ёмкости получаем: комплексное сопротивление (ёмкостное сопротивление)

(8.14)

и комплексную проводимость (ёмкостную проводимость)

(8.15)

Выводы:

комплексные сопротивление (8.14) и проводимость (8.15) ёмкости имеют только реактивные составляющие:

поэтому элемент ёмкости также называют реактивным.

гармоническое напряжение на ёмкости отстаёт оттока на поскольку

что следует из (8.11), т.е. ток и напряжение находятся в квадратуре (рис. 8.1, в);

значение средней мощности в элементе ёмкости так же, как и в индуктивности, равно нулю:

это объясняется тем, что в элементе ёмкости энергия не рассеивается; в режиме гармонических колебаний происходит обмен энергией между ёмкостью и подключённой к ней внешней цепью.

Комплексные сопротивления и проводимости двухполюсников

Проиллюстрируем вычисления комплексных сопротивлений и проводимостей на простейших примерах последовательного соединения резистивного элемента с индуктивным (рис. 8.2, а) и ёмкостным (рис. 8.2, б).

Последовательное соединение резистивного и индуктивного элементов

Алгебраическая форма записи комплексного сопротивления рассматриваемого двухполюсника (рис. 8.2, а)

(8.16)

где активная составляющая и реактивная составляющая

Полное сопротивление двухполюсника равно

(8.17)

поэтому показательная форма записи комплексного сопротивления имеет вид

(8.18)

Комплексная проводимость по определению для данного двухполюсника такова:

Найдём активную и реактивную части комплексной проводимости, для чего умножим числитель и знаменатель полученного выражения на комплексное число, сопряжённое знаменателю, а затем выделим вещественную и мнимую составляющие:

Отсюда модуль и аргумент комплексной проводимости соответственно равны:

(8.19)

(8.20)

и, наконец, для показательной формы комплексной проводимости получаем:

(8.21)

Последовательное соединение резистивного и ёмкостного элементов

Алгебраическая форма записи комплексного сопротивления рассматриваемого двухполюсника (рис. 8.2, б)

(8.22)

Полное сопротивление двухполюсника равно:

(8.24)

показательная форма имеет вид:

(8.25)

Комплексная проводимость по определению для данного двухполюсника такова:

B полученном выражении в силу равенства имеем:

поэтому
(8.26)

Из (8.26) получаем полную проводимость и аргумент двухполюсника соответственно:

(8.27)

(8.28)

Наконец, найдём активную и реактивную части комплексной проводимости:

(8.29)

Выводы:

Реактивные составляющие сопротивления и проводимости пассивных двухполюсников могут иметь как положительные, так и отрицательные значения;

  • если, то говорят, что сопротивление двухполюсника имеет индуктивный характер (на входе двухполюсника колебания напряжения опережают по фазе колебания тока); при этом на частоте сопротивление двухполюсника является чисто активным и равным R,поскольку сопротивление элемента индуктивности при постоянном токе равно нулю, т. е. индуктивность представляет собой короткое замыкание, а при сопротивление двухполюсника стремится к поскольку сопротивление элемента индуктивности стремится к бесконечности, т. е. индуктивность представляет собой разрыв цепи;
  • если же , то говорят, что сопротивление двухполюсника имеет ёмкостной характер (на входе двухполюсника колебания напряжения отстают по фазе от колебаний тока); при этом на частоте сопротивление двухполюсника стремится к поскольку сопротивление ёмкости стремится к бесконечности, т. е. ёмкость представляет собой разрыв цепи; а при сопротивление двухполюсника становится равным R, поскольку сопротивление ёмкости стремится к нулю, т. е. ёмкость представляет собой короткое замыкание.

Анализ установившихся гармонических колебаний в простейших цепях

Определения режимов состояния электрической цепи:

Колебания в цепях, имеющих реактивные элементы, качественно отличаются от колебаний, происходящих в резистивных цепях. Причиной качественных отличий является способность реактивных элементов выступать как в роли потребителя энергии, чему соответствуют положительные значения мгновенной мощности на зажимах элемента, так и в роли источника, когда элемент отдаёт накопленную энергию в цепь, чему соответствуют отрицательные значения мгновенной мощности на зажимах элемента. Процессы накопления и возврата энергии реактивными элементами не могут прекратиться и начаться сразу же после окончания внешних воздействий на цепь. Колебания в цепи продолжаются за счёт накопленной в реактивных элементах энергии, т. е. цепь обладает электромагнитной инерцией. Характер колебаний зависит от вида воздействия, схемы цепи, наличия начального запаса энергии в реактивных элементах к моменту приложения воздействия и т. д.

Колебания в цепях разделяют на установившиеся (стационарные) и неустановившиеся (нестационарные).

Колебания считаются установившимися, если все напряжения и токи в цепи изменяются как периодические функции времени с периодом Т, т. е. когда

Частным случаем периодических колебаний являются гармонические напряжения и токи.

Режим гармонических колебаний относится к числу установившихся режимов колебаний.

Режимом постоянного тока называется такое состояние цепи, в котором значения всех напряжений и токов не изменяются во времени:

Режимом покоя, или нулевыми начальными условиями называется такое состояние цепи, в котором значения всех напряжений и токов равны нулю.

Режимом переходных колебаний, или переходным процессом называется такое состояние цепи, в котором происходит переход из одного установившегося режима в другой установившийся режим. Режим переходных колебаний принадлежит к неустановившимся режимам.

Переходным временем называется время перехода из одного установившегося режима в другой установившийся режим.

Здесь и далее, если это не будет оговорено особо, рассматриваются цепи, находящиеся в режиме гармонических колебаний.

Анализ линейной цепи в режиме гармонических колебаний методом комплексных амплитуд состоит в следующем:

1. Гармонические токи и напряжения заменяются их комплексными изображениями: комплексными амплитудами или комплексными действующими значениями

(8.30)

2. Составляются уравнения (системы уравнений) для комплексных изображений токов и напряжений согласно законам Ома и Кирхгофа.

3. Решаются уравнения (системы уравнений) относительно комплексных изображений требуемых токов и напряжений.

4. Осуществляется переход от комплексных изображений токов и напряжений к их оригиналам.

Анализ гармонических колебаний в последовательном RL-контуре

Задача 8.1.

Найти напряжения и токи в последовательном контуре, изображённом на рис. 8.3.

Решение. Как было показано ранее, такой контур обладает комплексным сопротивлением

Комплексная амплитуда тока в контуре согласно закону Ома равна:

где — комплексная амплитуда напряжения источника гармонических колебаний. По определению комплексной амплитуды тока её модуль равен амплитуде, а её аргумент — начальной фазе гармонического тока в контуре. Отсюда имеем:

(8.31)

Определим комплексные амплитуды напряжений на элементах контура:

Отсюда для оригиналов напряжений имеем:

(8.32)

(8.33)

амплитуда тока в контуре зависит не только от значений индуктивности и сопротивления, но и от частоты гармонического воздействия (читателю предлагается самостоятельно оценить, что происходит в контуре при и )

колебания напряжения на входе контура опережают по фазе колебания тока в контуре на угол что объясняется индуктивным характером сопротивления контура, т. е. ток отстаёт по фазе от напряжения на контуре;

колебания напряжения на резистивном элементе происходят в фазе с колебаниями тока в контуре и отстают по фазе на угол от колебаний напряжения источника;

колебания напряжения на индуктивности опережают по фазе колебания напряжения источника на угол
и колебания тока в контуре на угол

Анализ гармонических колебаний в RLC-контуре

Задача 8.2.

Найти напряжения и токи в RLC-контуре, изображённом на рис. 8.4, а.

1. Определим эквивалентную комплексную проводимость контура (рис. 8.4,6)

2. Вычислим комплексную амплитуду напряжения на зажимах двухполюсника

где — комплексная амплитуда задающего тока источника и — комплексная амплитуда напряжения на ёмкости.

3. Найдём комплексные амплитуды токов в ветвях контура

4. Последние формулы позволяют записать выражения для комплексных амплитуд напряжений на элементах индуктивности и сопротивления:

Амплитуды и начальные фазы колебаний можно найти, представив комплексные амплитуды колебаний в показательной форме, что предлагается выполнить читателю.

Анализ сложных линейных электрических цепей в режиме установившихся гармонических колебаний

Ранее было показано (см. разд. 7.3), что комплексные амплитуды колебаний можно найти из решения систем уравнений Кирхгофа, узловых или контурных уравнений. Поэтому при составлении систем уравнений для комплексных амплитуд необходимо пользоваться правилами, установленными для резистивных цепей. Отличие будет состоять лишь в формальной замене обозначений сопротивлений и проводимостей на обозначения комплексных сопротивлений и проводимостей, а токи и напряжения заменить их комплексными амплитудами. Для удобства обозначений при составлении систем уравнений принято вместо комплексных амплитуд и использовать комплексные действующие значения колебаний (8.30); комплексные сопротивления и проводимости обозначают как Z и Y соответственно. При этом сами комплексные действующие значения токов и напряжений называют просто токами и напряжениями, если это не приводит к недоразумениям.

При этих обозначениях имеем канонические формы записи системы уравнений для комплексных узловых напряжений согласно (5.2)

(8.34)

и системы контурных уравнении для комплексных контурных токов согласно (5.9)

(8.35)

Перед решением задачи анализа гармонических колебаний символическим методом целесообразно сначала найти комплексные проводимости или сопротивления двухполюсников, составляющих ветви цепи, и только после этого составлять систему уравнений. При этом граф цепи упрощается и уменьшается число независимых уравнений.

Пример 8.1.

Рассмотрим схему цепи, изображённую на рис. 8.5, а. В схеме выделены три двухполюсника с сопротивлениями которые нетрудно найти по правилам последовательного и параллельного соединения элементов. Такое преобразование позволило свести исходную схему к эквивалент

Для схемы (рис. 8.5, б) нетрудно составить систему контурных уравнений:

Из этой системы легко получить последовательно:

значения комплексных контурных токов,

значения комплексных напряжений на комплексных сопротивлениях и на резисторе R,

величины напряжений на всех элементах схемы согласно разд. 8.2.2.

Особенности составления уравнений цепей с индуктивными связями

До сих пор рассматривались цепи, не содержащие индуктивно связанных элементов. Однако в реальных цепях широко используются трансформаторы, предназначенные для преобразования значений переменных напряжений и токов.

Основные соотношения

Простейший воздушный трансформатор без потерь (рис. 8.6) состоит из двух индуктивно связанных элементов индуктивности и .

Напряжения и токи на внешних зажимах этих индуктивностей связаны соотношениями:

(8.36)

где М — взаимная индуктивность между элементами и , равная

Коэффициент к называется коэффициентом связи; он характеризует степень магнитной связи между элементами и . Связь при называется жёсткой: весь магнитный поток, сцепляющийся с витками одной индуктивности, сцепляется с витками другой; значение при соответствует отсутствию связи.

Знаки в равенствах (8.36) зависят от направлений магнитных потоков в индуктивностях, а сами магнитные потоки зависят от направлений токов, проходящих через индуктивности. На схемах зажимы индуктивностей, через которые положительные частицы проходят в одном и том же направлении (к индуктивности или от неё), помечаются точками. Такие зажимы (узлы) называются одноимёнными. Одинаково ориентированные относительно одноимённых узлов токи создают складывающиеся потокосцепления. Поскольку в задачах анализа направления токов в индуктивностях выбираются независимо и произвольно, различают согласное и встречное направления отсчётов токов и напряжений. В уравнениях (8.36) согласному направлению соответствует знак “+”, а встречному — знак “-“. Варианты согласного и встречного выбора направлений отсчётов токов представлены на рис. 8.7.

Метод развязки индуктивных связей

Для составления уравнений цепи, содержащей индуктивные связи, используют такие схемы их замещения, в которых индуктивные связи отсутствуют. Метод, приводящий к таким схемам замещения, называют методом развязки индуктивных связей.

Рассмотрим наиболее важный для практики случай, когда взаимодействующие катушки имеют один общий узел (рис. 8.8, а). Любая схема замещения, исходя из (8.36), составляется только из элементов индуктивности, число которых должно равняться как минимум трём, поскольку уравнения содержат три коэффициента:

Воспользуемся схемой замещения рис. 8.8, б, для которой запишем систему контурных уравнений:

(8.37)

Полученная система не будет отличаться от системы (8.36) при условии:

(8.38)

Таким образом, схема рис. 8.8, б является схемой замещения двух связанных магнитным потоком индуктивностей, если значения элементов этой схемы равны:

(8.39)

В формулах (8.39) следует выбирать нижние знаки лишь в том случае, когда только один из двух соединённых в узел зажимов цепи рис. 8.8, а помечен точкой. В других случаях необходимо выбирать нижние знаки. Полученная схема называется Т-образной схемой замещения.

при жёсткой связи, когда и, следовательно, имеем:

откуда после приведения подобных членов получаем, что значения индуктивностей Т-образной схемы замещения удовлетворяют соотношению

(8.40)

которое может выполняться, если одна из индуктивностей схемы замещения является отрицательной. Если связь не является жёсткой, т. е. равенство (8.40) переходит в неравенство

что также не исключает возможности появления отрицательной индуктивности. На пассивных элементах отрицательная индуктивность физически не осуществима, однако её наличие в схеме замещения не противоречит задаче анализа колебаний в цепи и способствует решению этой задачи.

Применяется также и другая схема замещения (рис. 8.8, в), называемая П-образной. Соотношения между элементами исходной схемы (рис. 8.8, a) и схемы замещения

(8.41)

можно найти, если для рис. 8.8, в составить систему из двух узловых уравнений. Знаки в этих формулах выбираются по тому же правилу, что и в (8.39). В рассмотренной схеме замещения также возможно появление одной отрицательной индуктивности.

Символический метод расчета электрических цепей переменного тока

Действия над комплексными числами:

Символический метод нашел широкое применение для расчета сложных цепей переменного тока.

Символический метод расчета основан на использовании комплексных чисел.

Комплексное число А состоит из вещественной и мнимой частей, т. е.

Комплексное число на комплексной плоскости можно представить вектором. Проекция вектора на вещественную ось (ось абсцисс) соответствует вещественной части комплексного числа (рис. 14.1а). Проекция вектора на мнимую ось j (ось ординат) соответствует коэффициенту при мнимой единице . Мнимая единица у представляет собой поворотный множитель, умножение на который означает поворот вектора на 90° против часовой стрелки, т.е. в положительном направлении. Мнимая единица Тогда

Комплексным числам соответствуют векторы изображенные на комплексной плоскости (рис. 14.1а и б) в масштабе.

Модуль комплексного числа соответствует длине вектора, изображающего это комплексное число.

Из построения (рис. 14.1а) видно, что модули комплексных чисел определяются выражением

Следовательно,

Углы образованные векторами с положительным направлением вещественной оси, называются аргументами комплексного числа.

Аргументы комплексного числа (рис. 14.1а) определяются выражением

То есть

Как видно, аргумент комплексного числа отрицательный, так как вектор повернут на угол по часовой стрелке, а не против.

Существует три формы записи комплексного числа:

1) алгебраическая:

2)тригонометрическая:

так как

3) показательная:

где – основание натурального логарифма, однако в данном случае имеет чисто символическое значение.

Для перевода из показательной формы записи комплексного числа в алгебраическую пользуются тригонометрической формой записи комплексного числа (14.4).

Для перевода из алгебраической формы записи комплексного числа в показательную определяют модуль по (14.1) и аргумент по (14.2) комплексного числа.

Для перевода комплексного числа из одной формы в другую можно использовать логарифмическую линейку или микрокалькулятор.

Комплексные числа можно складывать, вычитать, умножать и делить.

Сложение и вычитание комплексных чисел производится только в алгебраической форме

На рис. 14.16 видно, что сложение и вычитание комплексных чисел соответствует сложению и вычитанию векторов, изображающих эти числа.

Умножение и деление комплексных чисел можно производить 5 алгебраической форме:

Для того чтобы избавиться от комплексов в знаменателе, числитель и знаменатель умножают на комплекс, сопряженный с комплексом знаменателя. У сопряженного комплекса знак перед мнимой единицей изменяется на обратный.

Произведение двух сопряженных комплексов — вещественное число, равное сумме квадратов вещественной и мнимой частей этих комплексов.

Однако умножение и деление комплексных чисел удобно производить в показательной форме.

При умножении комплексных чисел в показательной форме модули этих чисел перемножаются, а аргументы складываются алгебраически:

При делении комплексных чисел в показательной форме модули этих чисел делятся, а аргументы вычитаются с учетом знаков:

Таким образом, сложение и вычитание комплексных чисел можно производить только в алгебраической форме, а умножение и деление удобней и проще производить в показательной форме.

Ток, напряжение и сопротивление в комплексном виде

Если ток и напряжение изменяются по синусоидальному закону то, как указывалось выше, их можно изобразить векторами и, следовательно, записать комплексными числами:

где – комплексы тока и напряжения. Точка над комплексами указывает, что ток и напряжение изменяются по синусоидальному закону с определенной частотой — модули комплексов тока и напряжения, они же действующие значения тока и напряжения — аргументы комплексов тока и напряжения, они же начальные фазы тока и напряжения

Для неразветвленной цепи с (рис. 12.1а) мгновенные значения синусоидального тока и напряжения можно записать так: Тогда комплексы тока и напряжения

Комплекс полного сопротивления цепи определяется отношением комплекса напряжения к комплексу тока, т. е.

Комплексные величины, не зависящие от времени, обозначаются прописными буквами с черточкой внизу.

Модулем комплекса полного сопротивления является кажущееся сопротивление цепи а аргументом — угол сдвига фаз между током и напряжением

Алгебраическая форма записи комплекса полного сопротивления

Вещественная часть комплекса полного сопротивления есть активное сопротивление R, а коэффициент при мнимой единице j -реактивное сопротивление X. Знак перед поворотным множителем (мнимой единицей) указывает на характер цепи. Знак «плюс» соответствует цепи индуктивного характера, а знак «минус» – цепи емкостного характера.

Выражения комплексов сопротивлений различных цепей приедены в Приложении 7.

Обратная величина комплекса сопротивления — комплекс проводимости

Любую цепь переменного тока можно рассчитывать по заколам постоянного тока, если все величины представить в комплексной форме. В этом и заключается достоинство символического года расчета.

Мощность в комплексном виде

Для неразветвленной цепи с (рис. 12.3а) мгновенные значения тока и напряжения можно записать как

Комплексы напряжения и тока соответственно равны

Комплекс полной мощности цепи определяется произведением комплекса напряжения и сопряженного комплекса тока (над сопряженным комплексом синусоидальной величины ставят «звёздочку»)

Таким образом, модулем комплекса полной мощности является кажущаяся мощность цепи а аргументом — угол сдвига фаз между током и напряжением.

Если комплекс полной мощности перевести из показательной формы в алгебраическую, то получится

То есть вещественная часть комплекса полной мощности — активная мощность Р, а коэффициент при мнимой единице — реактивная мощность Q.

Знак перед поворотным множителем j указывает на характер цепи. В рассматриваемой цепи реактивная мощность емкостного характера

Комплексы величин токов, напряжений, сопротивлений, мощностей и других параметров цепи синусоидального тока необходимо выражать в двух видах записи комплексного числа: показательной и алгебраической. В этом случае сразу определяются действующие значения тока, напряжения, кажущееся сопротивление, его активные и реактивные части угол сдвига фаз между током и напряжением, характер цепи, кажущаяся S, активная Р и реактивная Q мощности. Кроме того, в неразветвленной цепи напряжения на участках складываются, суммируются токи в разветвленных цепях, а сложение комплексов можно производить только в алгебраической форме записи. В алгебраической форме записи кажущейся мощности сразу определяются активная мощность Р и реактивная мощность Q. В показательной форме записи сопротивлений производится их умножение и деление, необходимое при расчете цепей синусоидального тока при смешанном соединении потребителей, и т.д. Необходимость выражения комплексов в двух видах следует из примеров, разобранных в этой главе.

Пример 14.1

Для цепи, изображенной на рис. 14.2а, дано:

Определить токи напряжение на участках мощности S, Р и Q цепи; угол и характер цепи.

Построить векторную диаграмму цепи.

Решение

Комплексы сопротивлений участков (по номерам токов) и полного сопротивления цепи будут равны

Комплекс сопротивления участка CD цепи:

Тогда полное сопротивление цепи равно

Вектор заданной величины (тока или напряжения) можно направить в любом направлении. Однако удобнее совмещать его с вещественной или мнимой осью.

В рассмотренном примере заданное напряжение направляется по вещественной оси. Таким образом, комплекс общего напряжения будет равен

Комплекс тока цепи равен комплексу первого тока

.

Комплекс напряжения на участке АС:

Комплекс напряжений на участке CD:

Комплексы токов

Комплекс полной мощности цепи:

Из расчета цепи (рис. 14.2а) символическим методом следует:

Характер цепи емкостной, так как угол отрицательный. Векторная диаграмма для рассматриваемой цепи с учетом начальных фаз напряжений и токов изображена на рис. 14.2б.

Пример 14.2

Для цепи, изображенной на рис. 14.3, дано:

Определить токи напряжение цепи ; угол и характер цепи.

Решение

Комплексы сопротивлений участков (по номерам токов):

Вектор заданного тока в примере направим по мнимой оси, т. е.

Комплекс напряжения на участке СD:

Значение токов будут равны соответственно

Комплекс напряжения на участке АС:

Комплекс напряжения на участке АВ, т. е. напряжение сети, равен

Комплекс тока

Комплекс тока цепи:

Комплекс полной мощности цепи:

Характер цепи емкостной.

Пример 14.3

По условиям примера 14.2 определить полное сопротивление цепи (рис. 14.3).

Решение


Результаты расчета: полное сопротивление цепи (рис. 14.3) угол сдвига фаз характер цепи – емкостной

Погрешность 10′ при расчете угла в примерах 14.2 и 14.3 в пределах допустимого.

Рекомендую подробно изучить предметы:
  1. Электротехника
  2. Основы теории цепей
Ещё лекции с примерами решения и объяснением:
  • Четырехполюсники
  • Линейные диаграммы
  • Круговые диаграммы
  • Цепи с взаимной индукцией
  • Линейные электрические цепи
  • Нелинейные электрические цепи
  • Магнитные цепи и их расчёт
  • Цепи переменного тока

При копировании любых материалов с сайта evkova.org обязательна активная ссылка на сайт www.evkova.org

Сайт создан коллективом преподавателей на некоммерческой основе для дополнительного образования молодежи

Сайт пишется, поддерживается и управляется коллективом преподавателей

Whatsapp и логотип whatsapp являются товарными знаками корпорации WhatsApp LLC.

Cайт носит информационный характер и ни при каких условиях не является публичной офертой, которая определяется положениями статьи 437 Гражданского кодекса РФ. Анна Евкова не оказывает никаких услуг.

[spoiler title=”источники:”]

http://www.ups-info.ru/for_partners/library/teoreticheskie_osnove_ilektrotehniki_dlya_ibp_ups_/osnove_simvolicheskogo_metoda_rascheta_metode_kont/

http://www.evkova.org/simvolicheskij-metod-rascheta-tsepej

[/spoiler]


Загрузить PDF


Загрузить PDF

Полное сопротивление, или импеданс, характеризует сопротивление цепи переменному электрическому току. Данная величина измеряется в омах. Для вычисления полного сопротивления цепи необходимо знать значения всех активных сопротивлений (резисторов) и импеданс всех катушек индуктивности и конденсаторов, входящих в данную цепь, причем их величины меняются в зависимости от того, как меняется проходящий через цепь ток. Импеданс можно рассчитать при помощи простой формулы.

Формулы

  1. Полное сопротивление Z = R или XLили XC (если присутствует что-то одно)
  2. Полное сопротивление (последовательное соединение) Z = √(R2 + X2) (если присутствуют R и один тип X)
  3. Полное сопротивление (последовательное соединение) Z = √(R2 + (|XL – XC|)2) (если присутствуют R, XL, XC)
  4. Полное сопротивление (любое соединение) = R + jX (j — мнимое число √(-1))
  5. Сопротивление R = I / ΔV
  6. Индуктивное сопротивление XL = 2πƒL = ωL
  7. Емкостное сопротивление XC = 1 / 2πƒL = 1 / ωL
  1. Изображение с названием Calculate Impedance Step 1

    1

    Импеданс обозначается символом Z и измеряется в омах (Ом). Вы можете измерить импеданс электрической цепи или отдельного элемента. Импеданс характеризует сопротивление цепи переменному электрическому току. Есть два типа сопротивления, которые вносят вклад в импеданс:[1]

    • Активное сопротивление (R) зависит от материала и формы элемента. Наибольшим активным сопротивлением обладают резисторы, но и другие элементы цепи обладают небольшим активным сопротивлением.
    • Реактивное сопротивление (X) зависит от величины электромагнитного поля. Наибольшим реактивным сопротивлением обладают катушки индуктивности и конденсаторы.
  2. Изображение с названием Calculate Impedance Step 2

    2

    Сопротивление — это фундаментальная физическая величина, описываемая законом Ома: ΔV = I * R.[2]
    Эта формула позволит вам вычислить любую из трех величин, если вы знаете две другие. Например, чтобы вычислить сопротивление, перепишите формулу так: R = I / ΔV. Вы также можете измерить сопротивление при помощи мультиметра.

    • ΔV — это напряжение (разность потенциалов), измеряемое в вольтах (В).
    • I — сила тока, измеряемая в амперах (А).
    • R — это сопротивление, измеряемое в омах (Ом).
  3. Изображение с названием Calculate Impedance Step 3

    3

    Реактивное сопротивление имеет место только в цепях переменного тока. Как и активное сопротивление, реактивное сопротивление измеряется в омах (Ом). Есть два типа реактивного сопротивления:

    • Индуктивным сопротивлением XC обладают катушки индуктивности, создающие магнитное поле, которое препятствует изменению направления тока в цепи.[3]
      Чем быстрее меняется направление тока, тем больше индуктивное сопротивление.
    • Емкостным сопротивлением XC обладают конденсаторы, которые накапливают электрический заряд. При изменении направления тока в цепи конденсатор неоднократно обнуляет и накапливает электрический заряд. Чем дольше конденсатор заряжается, тем больше емкостное сопротивление.[4]
      Поэтому чем быстрее меняется направление тока, тем меньше емкостное сопротивление.
  4. Изображение с названием Calculate Impedance Step 4

    4

    Вычислите индуктивное сопротивление. Это сопротивление прямо пропорционально быстроте изменения направления тока, то есть частоты тока. Эта частота обозначается символом ƒ и измеряется в герцах (Гц). Формула для расчета индуктивного сопротивления: XL = 2πƒL, где L — индуктивность, измеряемая в генри (Гн).[5]

    • Индуктивность L зависит от количества витков в катушке индуктивности.[6]
      Также вы можете измерить индуктивность.
    • Если вы знакомы с единичной окружностью, то представьте, что один цикл переменного тока равен одному полному вращению этой окружности (на 2π радиан). Если умножить это значение на ƒ, которая измеряется в герцах (единиц в секунду), вы получите результат, измеряемый в радианах в секунду. Это единица измерения угловой скорости, которая обозначается через ω. Вы можете переписать формулу для вычисления индуктивного сопротивления так: XL=ωL[7]
  5. Изображение с названием Calculate Impedance Step 5

    5

    Вычислите емкостное сопротивление. Это сопротивление обратно пропорционально быстроте изменения направления тока, то есть частоты тока. Формула для вычисления емкостного сопротивления: XC = 1 / 2πƒC.[8]
    С — это емкость конденсатора, измеряемая в фарадах (Ф).

    • Вы можете измерить электрическую емкость.
    • Эту формулу можно переписать так: XC = 1 / ωL (объяснения см. выше).

    Реклама

  1. Изображение с названием Calculate Impedance Step 6

    1

    Если цепь состоит исключительно из резисторов, то импеданс вычисляется следующим образом. Сначала измерьте сопротивление каждого резистора или посмотрите значения сопротивления на схеме цепи.[9]

    • Если резисторы соединены последовательно, то полное сопротивление R = R1 + R2 + R3
    • Если резисторы соединены параллельно, то полное сопротивление R = 1 / R1 + 1 / R2 + 1 / R3
  2. Изображение с названием Calculate Impedance Step 7

    2

    Сложите одинаковые реактивные сопротивления. Если в цепи присутствуют исключительно катушки индуктивности или исключительно конденсаторы, то полное сопротивление равно сумме реактивных сопротивлений. Вычислите его следующим образом:[10]

    • Последовательное соединение катушек: Xtotal = XL1 + XL2 + …
    • Последовательное соединение конденсаторов: Ctotal = XC1 + XC2 + …
    • Параллельное соединение катушек: Xtotal = 1 / (1/XL1 + 1/XL2 …)
    • Параллельное соединение конденсаторов: Ctotal = 1 / (1/XC1 + 1/XC2 …)
  3. Изображение с названием Calculate Impedance Step 8

    3

    Вычтите индуктивные и емкостные сопротивления, чтобы получить общее реактивное сопротивление. Так как при возрастании одного типа сопротивления другое уменьшается, то они, как правило, компенсируют друг друга. Чтобы найти общее реактивное сопротивление, вычтите меньшее сопротивление из большего.[11]

    • Или воспользуйтесь формулой: Xtotal = |XC – XL|
  4. Изображение с названием Calculate Impedance Step 9

    4

    Вычислите импеданс по активному и реактивному сопротивлениям в последовательной цепи. Нельзя просто сложить эти величины, так как они меняются с течением времени, но достигают максимальных значений в разное время.[12]
    Поэтому воспользуйтесь формулой:Z = √(R2 + X2).[13]

    • Вычисления по этой формуле включают в себя использование векторов, но вы можете воспользоваться теоремой Пифагора, представив R и X в качестве катетов прямоугольного треугольника, а сопротивление Z — как гипотенузу.[14]
      [15]
  5. Изображение с названием Calculate Impedance Step 10

    5

    Вычислите импеданс по активному и реактивному сопротивлениям в параллельной цепи. В этом случае используются комплексные числа (это единственный способ вычислить полное сопротивление в параллельной цепи, в которой есть как активное, так и реактивное сопротивление).

    • Z = R + jX, где j — мнимая единица: √(-1). Используйте j вместо i, чтобы не перепутать мнимую единицу (j) с силой тока (I).
    • Складывать эти числа нельзя. Например, полное сопротивление может быть представлено так: 60 Ом + j120 Ом.
    • Если у вас есть две последовательные цепи, то вы можете отдельно сложить натуральные числа и отдельно — комплексные. Например, если Z1 = 60 Ом + j120 Ом, а к этой цепи последовательно подключен резистор с Z2 = 20Ω, то Ztotal = 80Ω + j120Ω.

    Реклама

Советы

  • Общее сопротивление (активное и реактивное сопротивления) также может быть выражено через мнимое число.

Реклама

Об этой статье

Эту страницу просматривали 168 342 раза.

Была ли эта статья полезной?

Автор статьи

Демьян Бондарь

Эксперт по предмету «Электроника, электротехника, радиотехника»

преподавательский стаж — 5 лет

Задать вопрос автору статьи

Основы комплексного расчета электрических цепей

Одним из основных способов расчета электрических цепей переменного тока является символический или комплексный метод. Как правило, он используется при анализе электрических схем с гармоническими токами, напряжениями и электродвижущей силой. В результате решения получается комплексное значение напряжений и токов. Синусоидальная величина может быть представлена:

  1. В форме вращающегося вектора.
  2. В виде комплексного числа.

Пример вращающегося вектора изображен на рисунке ниже.

Пример вращающегося вектора. Автор24 — интернет-биржа студенческих работ

Рисунок 1. Пример вращающегося вектора. Автор24 — интернет-биржа студенческих работ

Логотип iqutor

Сделаем домашку
с вашим ребенком за 380 ₽

Уделите время себе, а мы сделаем всю домашку с вашим ребенком в режиме online

Бесплатное пробное занятие

*количество мест ограничено

По данному рисунку видно, что синусоидальная величина а изменяется с течением времени, которая может быть входным напряжением или любым другим параметром электрической сети. Величина имеет некоторое начальное значение (t=0) при начальной фазе ф:

Формула. Автор24 — интернет-биржа студенческих работ

Рисунок 2. Формула. Автор24 — интернет-биржа студенческих работ

При угле Wt3, когда сумма Wt3+ф=90 и соответственно:

Формула. Автор24 — интернет-биржа студенческих работ

Рисунок 3. Формула. Автор24 — интернет-биржа студенческих работ

«Комплексный расчет электрических сетей» 👇

Синусоидальная величина при угле Wt7, когда сумма Wt7 + ф = 270 будет иметь отрицательное значение:

Формула. Автор24 — интернет-биржа студенческих работ

Рисунок 4. Формула. Автор24 — интернет-биржа студенческих работ

Величина будет иметь отрицательное значение при углах Wtn + ф = 0, когда Wtn = –ф (данная область на рисунке не отмечена), таким образом:

Формула. Автор24 — интернет-биржа студенческих работ

Рисунок 5. Формула. Автор24 — интернет-биржа студенческих работ

Также нулевое значение у синусоидальной величины будет при угле Wt11, когда Wt11+ ф = 360:

Формула. Автор24 — интернет-биржа студенческих работ

Рисунок 6. Формула. Автор24 — интернет-биржа студенческих работ

Именно по такому закону может меняться синусоидальная величина, например напряжение, изменяясь от 0 до максимального значения и обратно.

Другая форма представления — комплексная

Формула. Автор24 — интернет-биржа студенческих работ

Рисунок 7. Формула. Автор24 — интернет-биржа студенческих работ

Для этого строится график (комплексная плоскость) зависимости двух величин, как на рисунке ниже.

График. Автор24 — интернет-биржа студенческих работ

Рисунок 8. График. Автор24 — интернет-биржа студенческих работ

Длина вектора Аm равна максимальному значению амплитуды рассматриваемой величины. Если учитывать начальную фазу (ф), то это число записывается следующим образом.

Формула. Автор24 — интернет-биржа студенческих работ

Рисунок 9. Формула. Автор24 — интернет-биржа студенческих работ

В практических расчетах комплексного метода применяют не амплитудное значение, а действующее, которое меньше в корень из 2 амплитудного:

Формула. Автор24 — интернет-биржа студенческих работ

Рисунок 10. Формула. Автор24 — интернет-биржа студенческих работ

При работе с комплексными числами применяется один из трех способов записи комплексного числа: тригонометрическая форма, алгебраическая форма, показательная форма. Например, имеется комплексное число в показательной форме:

Формула. Автор24 — интернет-биржа студенческих работ

Рисунок 11. Формула. Автор24 — интернет-биржа студенческих работ

В тригонометрической форме оно будет иметь следующий вид:

Формула. Автор24 — интернет-биржа студенческих работ

Рисунок 12. Формула. Автор24 — интернет-биржа студенческих работ

В итоге при переходе в алгебраическую форму, учитывая, что:

Формула. Автор24 — интернет-биржа студенческих работ

Рисунок 13. Формула. Автор24 — интернет-биржа студенческих работ

получаем:

Формула. Автор24 — интернет-биржа студенческих работ

Рисунок 14. Формула. Автор24 — интернет-биржа студенческих работ

где, ReA = 8,66 – действительная составляющая комплексного числа; ImA = 5 – мнимая составляющая комплексного числа.

При переходе от алгебраической формы к показательной получаем число следующего вида

Формула. Автор24 — интернет-биржа студенческих работ

Рисунок 15. Формула. Автор24 — интернет-биржа студенческих работ

Оно переходит к показательной форме следующим преобразованием:

Формула. Автор24 — интернет-биржа студенческих работ

Рисунок 16. Формула. Автор24 — интернет-биржа студенческих работ

А угол рассчитывается по формуле

Формула. Автор24 — интернет-биржа студенческих работ

Рисунок 17. Формула. Автор24 — интернет-биржа студенческих работ

И в итоге получается:

Формула. Автор24 — интернет-биржа студенческих работ

Рисунок 18. Формула. Автор24 — интернет-биржа студенческих работ

Пример расчета электрической цепи комплексным методом

Алгоритм комплексного расчета электрической цепи выглядит следующим образом:

  1. Составляется комплексная схема электрической цепи, в которой мгновенные значения токов, напряжений и электродвижущей силы заменяются на комплексные.
  2. Выбираются и обозначаются направления токов.
  3. Составляется комплексное уравнение.
  4. Решается уравнение относительно комплексного значения искомой величины.
  5. При необходимости записываются мгновенные значения полученных комплексных величин.

Рассмотрим схему электрической цепи с последовательным соединением (рисунок ниже), в которой нам известны сопротивления — R1, R2 и R3, емкость — С, индуктивность — L и частота — f, электродвижущая сила е = 141sin*Wt (закон изменения ЭДС)

Схема электрической цепи. Автор24 — интернет-биржа студенческих работ

Рисунок 19. Схема электрической цепи. Автор24 — интернет-биржа студенческих работ

Сначала составляется комплексная схема, на которой обозначаются комплексные токи и напряжения.

Комплексная схема. Автор24 — интернет-биржа студенческих работ

Рисунок 20. Комплексная схема. Автор24 — интернет-биржа студенческих работ

Ток в цепи равен:

Формула. Автор24 — интернет-биржа студенческих работ

Рисунок 21. Формула. Автор24 — интернет-биржа студенческих работ

где, U – входное комплексное напряжение; Z – полное сопротивление всей электрической цепи.

Комплексное напряжение рассчитывается по формуле:

Формула. Автор24 — интернет-биржа студенческих работ

Рисунок 22. Формула. Автор24 — интернет-биржа студенческих работ

В данном случае начальная фаза ф=0, потому что для мгновенного значения напряжения вида

Формула. Автор24 — интернет-биржа студенческих работ

Рисунок 23. Формула. Автор24 — интернет-биржа студенческих работ

при ф =0 получаем:

Формула. Автор24 — интернет-биржа студенческих работ

Рисунок 24. Формула. Автор24 — интернет-биржа студенческих работ

Таким образом в показательной форме напряжение записывается:

Формула. Автор24 — интернет-биржа студенческих работ

Рисунок 25. Формула. Автор24 — интернет-биржа студенческих работ

Комплексное сопротивление имеет следующий общий вид:

Формула. Автор24 — интернет-биржа студенческих работ

Рисунок 26. Формула. Автор24 — интернет-биржа студенческих работ

Комплексное сопротивление емкости имеет следующий вид

Формула. Автор24 — интернет-биржа студенческих работ

Рисунок 27. Формула. Автор24 — интернет-биржа студенческих работ

Откуда общее комплексное сопротивление электрической сети рассчитывается по формуле:

Формула. Автор24 — интернет-биржа студенческих работ

Рисунок 28. Формула. Автор24 — интернет-биржа студенческих работ

Таким образом, после нахождения комплексного сопротивления становится возможным найти комплексный ток по второму закону Ома. Далее находятся комплексные напряжения на элементах цепи:

Формула. Автор24 — интернет-биржа студенческих работ

Рисунок 29. Формула. Автор24 — интернет-биржа студенческих работ

Для проверки результатов можно использовать второй закон Кирхгофа, согласно которому должно выполняться равенство:

Формула. Автор24 — интернет-биржа студенческих работ

Рисунок 30. Формула. Автор24 — интернет-биржа студенческих работ

В результате проверки допускается небольшое расхождение, которое может получиться в результате промежуточных округлений при расчете комплексных величин, а также преобразовании их из одной формы в другую.

Комплексный метод расчета электрических сетей уже давно доказал свою эффективность на практике. Его активно применяют на промышленных предприятиях, при строительстве объектов различного назначения, в том числе и объектах электроснабжения. Один из недостатков метода заключается в его большом объеме вычислительных и преобразовательных действий, поэтому для ускорения процесса используют электронно-вычислительные процессы и специальные программы.

Находи статьи и создавай свой список литературы по ГОСТу

Поиск по теме

Добавить комментарий